The Student Room Group

STEP II/III 2014 solutions

Scroll to see replies

Reply 40
STEP II Q3

i)

Spoiler


ii)
a)

Spoiler



b)

Spoiler



iii)

Spoiler

(edited 9 years ago)
Reply 41
Here is a solution to STEP III 2014 Q11
Original post by edrraa
Surely you can't equate real and imaginary parts when the numbers used are themselves complex?


Arghh that's true, but it works out since in both instances, the other part is always equal. I hope I wouldn't lose too many marks, I really need to keep as many as I can :frown:

I will edit the solution in a bit.
(edited 9 years ago)
Original post by Brammer
Here is a solution to STEP III 2014 Q2


did we actually have to use partial fractions in part i or could we not use the result from the formula book for the artanh sub integral?
STEP II - Q13

Spoiler

Reply 45
Original post by newblood
did we actually have to use partial fractions in part i or could we not use the result from the formula book for the artanh sub integral?


Yes - if the integral was provided in the formula book, then you could have used it without using partial fractions. Sorry if I complicated things unnecessarily.
STEP III - Q1


Spoiler

(edited 9 years ago)
Reply 47
Here is a solution to STEP II 2014 Q11
Reply 48
Original post by metaltron
Arghh that's true, but it works out since in both instances, the other part is always equal. I hope I wouldn't lose too many marks, I really need to keep as many as I can :frown:

I will edit the solution in a bit.

It's ok friend you are not alone in this feeling of regret and sadness.
STEP III, Q8:

For any 2 positive integers M < N, we have f(M) > f(M+1) > ... > f(N). The sum has (N+1-M) terms, all terms are >= f(N), and N-M terms are > f(N). So n=MNf(n)>n=MNf(N)=(N+1M)f(N)\sum_{n=M}^N f(n) > \sum_{n=M}^N f(N) = (N+1-M) f(N) and similarly MNf(n)<(M+1N)f(M)[\sum_M^N f(n) < (M+1-N) f(M)[.

Put M=kn,N=kn+11M = k^n, N = k^{n+1} - 1. (N+1M)=kn(k1)(N+1-M) = k^n(k-1), so as long as k >2 or n > 0 we have N+1-M > 1. (We can't prove strict inequality in the case n=0, k=2, but if you plug these in, we do actually get equality on the RHS, so it's not actually a strict inequality in this case, which is a minor error in the question).

But assuming we do have (N+1-M) > 1, then M < N, and so by the above we have kn(k1)f(kn+1)<r=knkn+11f(r)<kn(k1)f(kn)k^n(k-1)f(k^{n+1}) < \sum_{r=k^n}^{k^{n+1}-1} f(r) < k^n(k-1)f(k^n) as desired.

(i) Let f(n) = 1/n and k =2. Then we have 2n12n+1<r=2n2n+111r<2n12n\displaystyle 2^n \dfrac{1}{2^{n+1}} < \sum_{r=2^n}^{2^{n+1}-1} \frac{1}{r} < 2^n \dfrac{1}{2^n}, so 12<r=2n2n+111r<1\displaystyle \dfrac{1}{2} < \sum_{r=2^n}^{2^{n+1}-1} \frac{1}{r} < 1.

This gives us:

n=0N12<n=0Nr=2n2n+111r<n=0N1\displaystyle \sum_{n=0}^N \dfrac{1}{2} <\sum_{n=0}^N \sum_{r=2^n}^{2^{n+1}-1} \frac{1}{r} < \sum_{n=0}^N 1 and so N+12<12N+111r<N+1 \displaystyle \frac{N+1}{2} < \sum_1^{2^{N+1}-1} \frac{1}{r} < N+1 as desired.(note that again we actually get equality when N = 0).

(ii) Now take f(n) = 1/n^3, k = 2. We get r=2n2n+111r3<2n123n=1/4n\sum_{r=2^n}^{2^{n+1}-1} \frac{1}{r^3} < 2^n \dfrac{1}{2^{3n}} = 1/4^n and so

n=0Nr=2n2n+111r3<n=0N1/4n\sum_{n=0}^N \sum_{r=2^n}^{2^{n+1}-1} \frac{1}{r^3} < \sum_{n=0}^N 1/4^n so r=12N+111r3<0N(1/4)n<0(1/4)n=4/3\sum_{r=1}^{2^{N+1}-1} \frac{1}{r^3} < \sum_0^N (1/4)^n < \sum_0^\infty (1/4)^n = 4/3 and so finally r=11r3<4/3\sum_{r=1}^\infty \frac{1}{r^3} < 4/3.

(iii) Assume we write all integers 0, 1, ..., 999 using leading zeros (e.g. "009" for 9). Then S(1000){0} S(1000) \cup \{0\} is the set of all numbers where all 3 digits are NOT equal to 2. So there are 9 choices for each digit, so a total of 9^3 possibilities. Removing the "000" case (since 0 is not a positive integer) leaves 9^3 -1 as required.

Now for k>0 ρ(10k)ρ(10k1)\rho(10^{k})-\rho(10^{k-1}) is the sum of the reciprocals of all k digit numbers without a 2. There are 8 choices for the leading digit, 9 choices for each of the other k-1 digits, so there are 89k18 \cdot 9^{k-1} numbers of this form, and each number is at least 10k110^{k-1} (and at least one is larger). So ρ(10k)ρ(10k1)<89k1/(10k1)=8(910)k1\rho(10^{k})-\rho(10^{k-1}) < 8\cdot 9^{k-1}/(10^{k-1}) = 8\left(\frac{9}{10}\right)^{k-1}.

So ρ(10k)<81k(910)k1<8119/10=80\rho(10^k) < 8 \sum_1^k \left(\frac{9}{10}\right)^{k-1} < 8 \frac{1}{1-9/10} = 80. Since ρ(10k)ρ(n)\rho(10^k) \geq \rho(n) whenever 10^k > n, the result follows.

Note: I thought it more intuitive to do the end this way - it's probably not what they wanted, but it's essentially the same underneath. Also this was LaTeX hell, so there may well be typos.
(edited 9 years ago)
Original post by Khallil
II/4Maybe I'm missing something, but I don't see how you got that 01(1+u2)2du=π4\displaystyle \int_0^\infty \dfrac{1}{(1+u^2)^2}\,du = \dfrac{\pi}{4} (and this does seem to be only a linear substitution away from the original integral, so is dangerously close to circular, IMHO).
Original post by DFranklin
Maybe I'm missing something, but I don't see how you got that 01(1+u2)2du=π4\displaystyle \int_0^\infty \dfrac{1}{(1+u^2)^2}\,du = \dfrac{\pi}{4} (and this does seem to be only a linear substitution away from the original integral, so is dangerously close to circular, IMHO).


DJ pointed it out to me a short while ago. I'll edit it in a few minutes with the method that he recommended.
(edited 9 years ago)
Poincare's inequality in II Q2, Method of characteristics in II Q5 and (vaguely) calculus of variations in III Q4 :O - there must have been a PDE theorist on the STEP-setting committee this year, gutted I didn't get to answer them before other people!
(edited 9 years ago)
Step 2 question 1 solution
Original post by Stray
STEP II Q3

i)

Spoiler


ii)
a)

Spoiler



b)

Spoiler



iii)

Spoiler



I think (ii) (b) is a bit unsatisfactory....see file
Reply 55
Original post by mikelbird
I think (ii) (b) is a bit unsatisfactory....see file


Thanks - I've improved that. For some reason my brain had decided that the variables couldn't be separated... duh.
STEP II, Q6:

sin(r+1/2)x - sin(r-1/2)x = sin rx cos x/2 + cos rx sin x/2 - sin rx cos x/2 + cos rx sin x/2 = 2 cos rx sin x/2.

So 2sin(x/2) (cos x + cos 2x + ... + cos nx) = sin(n+1/2)x - sin(n-1/2)x + sin(n-1/2)x - sin(n-3/2)x + ... + sin x/2 - sin x/2. All terms on RHS except first and last cancel, and then dividing by 2sin x/2 (since we are given this is non zero) we get

1ncosrx=sin(n+12)xsin(12x)2sinx2 \displaystyle \sum_1^n \cos rx = \dfrac{\sin(n+\frac{1}{2})x - sin(\frac{1}{2}x)}{2\sin \frac{x}{2}} as desired.

S_2(x) = sin(x) + sin(2x) / 2, so ddxS2(x)=cosx+cos2x=2cos3x2cosx2\dfrac{d}{dx}S_2(x) = \cos x + \cos 2x = 2\cos\frac{3x}{2}\cos\frac{x}{2}. cos(x/2) is never zero for 0<=x<=pi, so the only root is where cos(3x/2) =0, i.e. 3x/2 = pi/2 + 2mpi, so x = pi/3 + 4/3m pi. If 0<=x<=pi the only possibilities is x = pi/3.

http://www5a.wolframalpha.com/Calculate/MSP/MSP2431d8ahdi9gcfi21ci000036di7dcag60dg904?MSPStoreType=image/gif&s=53&w=325.&h=150.&cdf=RangeControl

ddxsn(x)=1n1rrcosrx=1ncosrx=sin(n+12)xsin(12x)2sinx2\displaystyle \dfrac{d}{dx} s_n(x) = \sum_1^n \frac{1}{r} r \cos rx = \sum_1^n \cos rx = \dfrac{\sin(n+\frac{1}{2})x - sin(\frac{1}{2}x)}{2\sin \frac{x}{2}}

This has stationary points when sin(n+1/2)x - sin(1/2)x = 0. We can rewrite this as sin nx cos x/2 + cos nx sin x/2 - sin x/2 = 0, or sin(x/2)[(cos nx -1) + sin nx cot x / 2 ]= 0. We are given 0 < x < pi and so sin(x/2) is non-zero, so we can divde by it. Rearranging we get (1-cosnx) = sin nx cot x/2 and then multiplying by tan x/2 gives (1cosnx)tanx2=sinnx(1 -\cos nx)\tan \frac{x}{2} = \sin nx. So if x = x_0 is a stationary point we have (1cosnx0)tanx02=sinnx0(1-\cos n x_0) \tan \frac{x_0}{2} = \sin nx_0 as desired.

Note that since (1cosnx0)0(1-\cos n x_0)\geq 0 and tan(x0/2)0\tan(x_0/2)\geq 0 we must have sin(nx0)0\sin(nx_0) \geq 0 and so Sn(x0)Sn1(x0)S_n(x_0)\geq S_{n-1}(x_0) (since Sn(x0)=Sn1(x0)+1nsinnx0S_n(x_0) = S_{n-1}(x_0) + \frac{1}{n}\sin n x_0).

Suppose Sn1(x).>0S_{n-1}(x) .> 0 for all x in (0, pi). Now consider S_n. Since S_n(0)=S_n(pi) =0, S attains a minimum value at some point x_0 in (0, pi). Then by the argument above, Sn(x0)Sn1(x0)>0S_n(x_0) \geq S_n-1(x_0) > 0 as desired.

Finally, suppose (for contradiction) that we can find n1n \geq 1 and x[0,π]x\in[0,\pi] s.t. S_n(x) < 0. Wlog, we can choose the smallest such n. Note that n cannot be 1, since sin(x) >=0 for all x in [0,pi]. So n > 1, and since n is chosen minimal, we must have Sn10S_{n-1} \geq 0 on [0,pi]. But then in particular Sn1(x)0S_{n-1}(x) \geq 0 and so Sn(x)0S_n(x) \geq 0, contradiction.
Forgive the duplication of effort but I found the solution presented very difficult to read...
(edited 9 years ago)
STEP III - Q13

Spoiler

(edited 9 years ago)

Quick Reply

Latest

Trending

Trending